0 Daumen
176 Aufrufe

Aufgabeblob.png

Text erkannt:

Aufgabe 3: Gegeben sei die rekursive Folge \( \left(a_{n}\right)_{n \in \mathbb{N}_{0}} \) mit
\( a_{n+1}=\frac{a_{n}+5}{2} \text { sowie } a_{0}=1 . \)
Zeigen Sie durch vollständige Induktion, dass für alle \( n \in \mathbb{N}_{0} \) gilt:
\( a_{n}=5-\frac{4}{2^{n}} \)


Problem/Ansatz:

irgendwie komme ich leider nicht weiter.

I.A:

n=

a0= 1= 5- 4/2^n

   1= 5 - 4/2^0

   1= 5-4
     1= 1      w.A.

I.V. : Es existiert ein n element von N, sodass an= 5 - 4/2^n gilt.

I.S.:

n -> n+1

an+1= (an+5)/2

(I.V.)=((5-4/2^n)-5)/2


hier komme ich leider nicht weiter.

Über eine Antwort würde ich mehr sehr freuen.


Liebe Grüße

Text erkannt:

\( a_{n}=5-\frac{4}{2^{n}} \)

Avatar von

2 Antworten

0 Daumen

Setze an in die Rekursionsvorschrift ein: (Anwendung der Induktionsvoraussetzung)

an+1=\( \frac{5-\frac{4}{2^n}+5}{2} \) und vereinfache den Bruch.

an+1=5-\( \frac{4}{2^{n+1}} \).

Avatar von 123 k 🚀
0 Daumen

Hallo,

a_{n+1}=((5-4/2^n)+5)/2

=5/2 -4/2^{n+1} +5/2

=5-4/2^{n+1}

:-)

PS

Du hast hier

(I.V.)=((5-4/2^n)-5)/2

einen Vorzeichenfehler.

Avatar von 47 k

Ein anderes Problem?

Stell deine Frage

Willkommen bei der Mathelounge! Stell deine Frage einfach und kostenlos

x
Made by a lovely community